Physics for Scientists and Engineers: A Strategic Approach with Modern Physics (3rd Edition)

Published by Pearson
ISBN 10: 0321740904
ISBN 13: 978-0-32174-090-8

Chapter 26 - The Electric Field - Exercises and Problems - Page 777: 34

Answer

See the detailed answer below.

Work Step by Step

$$\color{blue}{\bf [a]}$$ First of all, we need to sketch the problem, where the dipole consists of two equla-unlike charges, as seen below. From the geometry of the figure below, we can see that: $\bullet$ $\theta_1=\theta_2=\theta$ $\bullet$ $r_1=r_2=\sqrt{x^2+(s/2)^2}$ $\bullet$ $\sin\theta=\dfrac{s/2}{r}=\dfrac{s}{2\sqrt{x^2+(s/2)^2}}$ $\bullet$ $\cos\theta=\dfrac{x}{r}=\dfrac{x}{\sqrt{x^2+(s/2)^2}}$ Thus, the electric field exerted by $q_1$ at a point at a distance of $x$, as seen below, is given by $$E_1=\dfrac{kq_1}{r_1^2}\left(\cos\theta\;\hat i-\sin\theta\;\hat j\right)$$ Plug the known; $$E_1=\dfrac{kq}{(x^2+(s/2)^2)}\left(\dfrac{x}{\sqrt{x^2+(s/2)^2}}\;\hat i-\dfrac{s}{2\sqrt{x^2+(s/2)^2}}\;\hat j\right)$$ $$E_1=\dfrac{kq}{\left(x^2+\dfrac{s^2}{4}\right)^{3/2}}\left(x\;\hat i-\dfrac{s}{2 }\;\hat j\right)\tag 1$$ The electric field by $q_2$ at the same point is given by $$E_2=\dfrac{kq_2}{r_2^2}\left(-\cos\theta\;\hat i-\sin\theta\;\hat j\right)$$ Plug the known; $$E_2=\dfrac{-kq}{(x^2+(s/2)^2)}\left(\dfrac{x}{\sqrt{x^2+(s/2)^2}}\;\hat i+\dfrac{s}{2\sqrt{x^2+(s/2)^2}}\;\hat j\right)$$ $$E_2=\dfrac{-kq}{\left(x^2+\dfrac{s^2}{4}\right)^{3/2}}\left(x\;\hat i+\dfrac{s}{2 }\;\hat j\right)\tag 2$$ Thus, the net electric field is given by $$E_{dipole}=E_1+E_2$$ Plug from (1) and (2), $$E_{dipole}=\dfrac{kq}{\left(x^2+\dfrac{s^2}{4}\right)^{3/2}}\left(x\;\hat i-\dfrac{s}{2 }\;\hat j\right)-\dfrac{kq}{\left(x^2+\dfrac{s^2}{4}\right)^{3/2}}\left(x\;\hat i+\dfrac{s}{2 }\;\hat j\right)$$ $$E_{dipole}=\dfrac{kq}{\left(x^2+\dfrac{s^2}{4}\right)^{3/2}}\left[ x\;\hat i-\dfrac{s}{2 }\;\hat j -( x\;\hat i+\dfrac{s}{2 }\;\hat j)\right]$$ $$ E_{dipole}= \dfrac{ -kqs }{\left(x^2+\dfrac{s^2}{4}\right)^{3/2}} \;\hat j\tag 3$$ And when $x$ is much greater than the length of the dipole $s$, $(x\gt \gt s)$, then $\left(x^2+\dfrac{s^2}{4}\right)^{3/2}\approx x^3$, Plug into (3), $$ E_{dipole}= \dfrac{ -kqs }{x^3} \;\hat j $$ Note that: - We can replace $r$ by $x$ for generalizing the term. - Recall that $\vec p=qs$ - Recall that $k=1/4\pi \epsilon_0$ $$ \boxed{\vec E_{dipole}=- \dfrac{ \vec p }{4\pi \epsilon_0 r^3} } $$ Where the direction of the elipole electric field, in this case, is in the opposite direction of the electric dipole moment.
Update this answer!

You can help us out by revising, improving and updating this answer.

Update this answer

After you claim an answer you’ll have 24 hours to send in a draft. An editor will review the submission and either publish your submission or provide feedback.